ISI MMath Solutions and Discussion 2018 : PMB

ISI MMath PMB 2018 Subjective Questions along with hints and solutions and discussions

Note: You must try the problems on your own before looking at the solutions. I have made the solutions so that you don’t have to join any paid program to get the solutions. After you have failed several times in your attempt to solve a question even with the hints provided, then only look at the solution to that problem. If you cannot solve a single question on your own then this exam is not for you. And if you like my work then please do share this among your friends and do comment below.  

To view ISI MMATH 2020 solutions, hints, and discussions: Click Here

To view ISI MMATH 2019 solutions, hints, and discussions: Click Here

To view ISI MMATH 2017 solutions, hints, and discussions: Click Here

To view ISI MMATH 2016 solutions, hints, and discussions: Click Here

To view previous year’s question papers: Click Here

Problem 1.

Find the values of a > 0 for which the improper integral

    \[ \int^\infty_0 \frac{\sin x}{x^a}dx \]

converges.

Topic: Real Analysis
Difficulty level: Medium

Indian Statistical Institute, ISI MMath 2018 Problem 1 Hints along with Solution & Discussion

Hint 1

The integral is defined as the limit of proper intergals

    \[ \int^\infty_0 \frac{\sin x}{x^a}dx=\lim_{\delta\to 0, C\to\infty}\int^C_{\delta}\frac{\sin x}{x^a}dx= \]

    \[ =\lim_{\delta\to 0}\int^1_\delta \frac{\sin x}{x^a}dx+\lim_{C\to \infty}\int^C_1\frac{\sin x}{x^a}dx \]

 

Hint 2

Using the relation \lim_{x\to 0}\frac{\sin x}{x}=1 we find \epsilon>0 such that for all x\in (0,\epsilon) we have \frac{1}{2}<\frac{\sin x}{x}<2.

Hint 3

Then for all \delta<\delta'<\epsilon we have a two-sided estimate

    \[ \left|\int^1_\delta \frac{\sin x}{x^a}dx-\int^1_{\delta'} \frac{\sin x}{x^a}dx\right|=\int^{\delta'}_\delta \frac{\sin x}{x^a}dx\leq 2 \int^{\delta'}_\delta x^{1-a}dx \]

and

    \[ \left|\int^1_\delta \frac{\sin x}{x^a}dx-\int^1_{\delta'} \frac{\sin x}{x^a}dx\right|=\int^{\delta'}_\delta \frac{\sin x}{x^a}dx\geq \frac{1}{2} \int^{\delta'}_\delta x^{1-a}dx \]

Full Solution

The integral is defined as the limit of proper intergals

    \[ \int^\infty_0 \frac{\sin x}{x^a}dx=\lim_{\delta\to 0, C\to\infty}\int^C_{\delta}\frac{\sin x}{x^a}dx= \]

    \[ =\lim_{\delta\to 0}\int^1_\delta \frac{\sin x}{x^a}dx+\lim_{C\to \infty}\int^C_1\frac{\sin x}{x^a}dx \]

 

Using the relation \lim_{x\to 0}\frac{\sin x}{x}=1 we find \epsilon>0 such that for all x\in (0,\epsilon) we have \frac{1}{2}<\frac{\sin x}{x}<2. Then for all \delta<\delta'<\epsilon we have a two-sided estimate

    \[ \left|\int^1_\delta \frac{\sin x}{x^a}dx-\int^1_{\delta'} \frac{\sin x}{x^a}dx\right|=\int^{\delta'}_\delta \frac{\sin x}{x^a}dx\leq 2 \int^{\delta'}_\delta x^{1-a}dx \]

and

    \[ \left|\int^1_\delta \frac{\sin x}{x^a}dx-\int^1_{\delta'} \frac{\sin x}{x^a}dx\right|=\int^{\delta'}_\delta \frac{\sin x}{x^a}dx\geq \frac{1}{2} \int^{\delta'}_\delta x^{1-a}dx \]

So, existence of the limit \lim_{\delta\to 0}\int^1_\delta \frac{\sin x}{x^a}dx is equivalent to finiteness of the integral \int^1_0 x^{1-a}dx. So,

    \[ \exists \lim_{\delta\to 0}\int^1_\delta \frac{\sin x}{x^a}dx\Leftrightarrow 1-a>-1\Leftrightarrow a<2. \]

Let 1<C<C'. We have an estimate

    \[ \left|\int^C_1 \frac{\sin x}{x^a}dx-\int^{C'}_1 \frac{\sin x}{x^a}dx\right|=\left| \int^{C'}_C\frac{\sin x}{x^a}dx\right|= \]

integrate by parts with u=x^{-a}, dv=\sin x dx, so that v=-\cos x, du=-ax^{-a-1}dx

    \[ =\left| \frac{\cos C}{C^a}-\frac{\cos C'}{(C')^a}-a\int^{C'}_C\frac{\cos x}{x^{a+1}}dx\right|\leq \]

    \[ \leq \frac{1}{C^a}+\frac{1}{(C')^a}+a\int^{\infty}_C\frac{dx}{x^{a+1}}=\frac{2}{C^a}+\frac{1}{(C')^a}\to 0, C,C'\to \infty. \]

So, the limit \lim_{C\to \infty}\int^C_1 \frac{\sin x}{x^a}dx exists for any a>0.

 

The improper integral \int^\infty_0 \frac{\sin x}{x^a}dx converges if and only if a<2.

 

Problem 2.

Let f : [0, 1] \to \mathbb{R} be a real-valued continuous function which is differentiable on (0, 1) and satisfies f(0) = 0. Suppose there exists a

constant c\in (0,1) such that

    \[ |f'(x)|\leq c|f(x)| \mbox{ for all } x\in (0,1). \]

Show that f(x) = 0 for all x\in [0,1] .

Topic: Real Analysis
Difficulty Level: Medium

Indian Statistical Institute, ISI MMath 2018 Problem 2 Hints along with Solution & Discussion

Hint 1

Take M=\max_{x\in (0,1)}|f(x)| and think of a way to bound |f(x)|.

Hint 2

For any x\in (0,1) we have

    \[ |f(x)|=|f(x)-f(0)|=\left|\int^x_0 f'(y)dy\right|\leq c \int^x_0 |f(y)|dy\leq cx M\leq cM. \]

Hint 3

Try to show that M=0.

Full Solution

Denote M=\max_{x\in (0,1)}|f(x)|. For any x\in (0,1) we have

    \[|f(x)|=|f(x)-f(0)|=\left|\int^x_0 f'(y)dy\right|\leq c \int^x_0 |f(y)|dy\leq cx M\leq cM.\]

Since the estimate holds for any x\in (0,1) we take maximum in x and get

    \[M\leq cM, \ (1-c)M\leq 0\]

The assumption 0<c<1 implies 0\leq M\leq 0, i.e. M=0 and f(x) = 0 for all x\in [0,1] .

Problem 3.

Let G be an abelian group of order n.

 

 

(a) If f : G \to\mathbb{C} is a function, then prove that for all h\in G,

    \[ \sum_{g\in G}f(g)=\sum_{g\in G}f(hg). \]

(b) Let \mathbb{C}^* be the multiplicative group of non-zero complex numbers and suppose f : G\to \mathbb{C}^* is a homomorphism. Prove that

    \[ \sum_{g\in G}f(g)=0 \mbox{ or } \sum_{g\in G}f(g)=n. \]

(c) If f : G \to \mathbb{C}^* is any homomorphism, then prove that

    \[ \sum_{g\in G}|f(g)|=n. \]

Topic: Group Theory
Difficulty Level: Medium

Indian Statistical Institute, ISI MMath 2018 Problem 3 Hints along with Solution & Discussion

Hint 1

(a) For any h\in G the mapping g\to hg is a bijection of G.

Hint 2

(b) For any h\in G we have

    \[f(h)\sum_{g\in G}f(g)=\sum_{g\in G}f(hg)=\sum_{g\in G}f(g)\]

Hint 3

(c) Identity

    \[|f(hg)|=|f(h)f(g)|=|f(h)||f(g)|\]

implies that g\to |f(g)| is also a homomorphism.

Full Solution

(a) For any h\in G the mapping g\to hg is a bijection of G. When g ranges through G, the product hg also ranges through G:

    \[ \sum_{g\in G}f(hg)=\sum_{g\in G}f(g). \]

(b) For any h\in G we have

    \[ f(h)\sum_{g\in G}f(g)=\sum_{g\in G}f(hg)=\sum_{g\in G}f(g) \]

Taking the sum in h\in G we get

    \[ \sum_{h\in G}f(h)\sum_{g\in G}f(g)=n\sum_{g\in G}f(g). \]

So,

    \[ \left(\sum_{g\in G}f(g)\right)^2=n\sum_{g\in G}f(g), \]

    \[ \sum_{g\in G}f(g)\left(n-\sum_{g\in G}f(g)\right)=0. \]

Either \sum_{g\in G}f(g)=0 or \sum_{g\in G}f(g)=n.

(c) Identity

    \[ |f(hg)|=|f(h)f(g)|=|f(h)||f(g)| \]

implies that g\to |f(g)| is also a homomorphism. By previous part, either

\sum_{g\in G}|f(g)|=0 or \sum_{g\in G}|f(g)|=n. For all g\in G we have f(g)\neq 0, |f(g)|>0 and \sum_{g\in G}|f(g)|>0. It follows that \sum_{g\in G}|f(g)|=n.

Problem 4.

(a) Is the ideal I = (X + Y, X - Y ) in the polynomial ring \mathbb{C}[X, Y ] a prime ideal? Justify your answer.

(b) Is the ideal I = (X + Y, X - Y ) in the polynomial ring \mathbb{Z}[X, Y ] a prime ideal? Justify your answer.

Topic: Ring Theory
Difficulty Level: Medium

Indian Statistical Institute, ISI MMath 2018 Problem 4 Hints along with Solution & Discussion

Hint 1

(a) Try to use the relation

    \[ X=\frac{1}{2}(X+Y)+\frac{1}{2}(X-Y), Y=\frac{1}{2}(X+Y)-\frac{1}{2}(X-Y) \]

Hint 2

Assume that I is a prime ideal in \mathbb{Z}[X,Y]. Consider polynomial P(X,Y)=2X=(X+Y)+(X-Y)\in I. Then either 2\in I or X\in I.

Hint 3

(b) Hint continued:

General element of I is of the form

    \[ (X+Y)Q(X,Y)+(X-Y)R(X,Y), \ Q(X,Y),R(X,Y)\in \mathbb{Z}[X,Y]. \]

Full Solution

(a) Relations

    \[ X=\frac{1}{2}(X+Y)+\frac{1}{2}(X-Y), Y=\frac{1}{2}(X+Y)-\frac{1}{2}(X-Y) \]

imply that I=(X,Y). General element of I is of the form XP(X,Y)+YQ(X,Y), i.e.

    \[ I=\{P\in \mathbb{C}[X,Y]: P(0,0)=0\}. \]

If PQ\in I, then P(0,0)Q(0,0)=0 which is possible only when P(0,0)=0 or Q(0,0)=0. I is a prime ideal.

(b) Assume that I is a prime ideal in \mathbb{Z}[X,Y]. Consider polynomial P(X,Y)=2X=(X+Y)+(X-Y)\in I. Then either 2\in I or X\in I. General element of I is of the form

    \[ (X+Y)Q(X,Y)+(X-Y)R(X,Y), \ Q(X,Y),R(X,Y)\in \mathbb{Z}[X,Y]. \]

In particular, an element of I vanished when X=Y=0. So, 2\not\in I. It follows that X\in I and

    \[ X=(X+Y)Q(X,Y)+(X-Y)R(X,Y)=X(Q(X,Y)+R(X,Y))+Y(Q(X,Y)-R(X,Y)). \]

Then, Q(X,Y)=R(X,Y) and

    \[ X=2Q(X,Y)X, 2Q(X,Y)=1, \]

which is impossible in \mathbb{Z}[X,Y]. I is not a prime ideal.

Problem 5.

Let n \geq 2 and A be an n\times n matrix with real entries. Let \mbox{Adj} A denote the adjoint of A, that is, the (i, j)-th entry of \mbox{Adj} A is the

(j, i)-th cofactor of A. Show that the rank of \mbox{Adj} A is 0, 1 or n.

Topic: Linear Algebra
Difficulty Level: Medium

Indian Statistical Institute, ISI MMath 2018 Problem 5 Hints along with Solution & Discussion

Hint 1

If \det A\ne 0, then the inverse of A is given by \mbox{Adj} A. In this case the rank of \mbox{Adj}A is n.

Hint 2

Assume that the rank of A is \leq n-2. Then each (n-1)\times (n-1) minor of A is zero, and all elements of \mbox{Adj} A are equal to zero. In this case the rank of \mbox{Adj} A is equal to zero.

Hint 3

Assume that the rank of A is equal to n-1. Then there exists a nonzero (n-1)\times (n-1) minor of A, so the rank of \mbox{Adj} A is \geq 1. Denote by a^{(ij)} the (i,j)-th cofactor of A. Then (\mbox{Adj} A)_{ij}=a^{(ji)}. Consider the product

A\cdot \mbox{Adj} A. Its (i,j)-th entry is

    \[ \sum^n_{k=1}a_{ik}( \mbox{Adj} A)_{kj}=\sum^n_{k=1}a_{ik}a^{(jk)}=0 \eqno(1) \]

Full Solution

If \det A\ne 0, then the inverse of A is given by \mbox{Adj} A. In this case the rank of \mbox{Adj}A is n.

 

Assume that the rank of A is \leq n-2. Then each (n-1)\times (n-1) minor of A is zero, and all elements of \mbox{Adj} A are equal to zero. In this case the rank of \mbox{Adj} A is equal to zero.

 

Assume that the rank of A is equal to n-1. Then there exists a nonzero (n-1)\times (n-1) minor of A, so the rank of \mbox{Adj} A is \geq 1. Denote by a^{(ij)} the (i,j)-th cofactor of A. Then (\mbox{Adj} A)_{ij}=a^{(ji)}. Consider the product

A\cdot \mbox{Adj} A. Its (i,j)-th entry is

    \[ \sum^n_{k=1}a_{ik}( \mbox{Adj} A)_{kj}=\sum^n_{k=1}a_{ik}a^{(jk)}=0 \eqno(1) \]

Indeed, if i=j then the sum in (1) is the determinant of A. If i\ne j, then the sum in (1) is the determinant of the matrix A after the j-th row is replaced with the i-th row. Such determinant is zero as the matrix has two identical rows. We see that A\cdot \mbox{Adj} A=0. That is, each column of \mbox{Adj} A is in the kernel of A. But the rank of A is n-1, hence the dimension of the kernel of A is 1 and the dimension of the column range of \mbox{Adj} A is \leq 1. This proves that the rank of \mbox{Adj} A is equal to 1.

Problem 6.

Suppose an urn contains a red ball and a blue ball. A ball is drawn at random and a ball of the same colour is added to the urn along with

the one that was drawn. This process is repeated indefinitely.

Let X denote the random variable that takes the value n if the first n - 1 draws yield red balls and the n-th draw yields a blue ball.

(a) If n\geq 1, find P(X > n).

(b) Show that the probability of a blue ball being chosen eventually is 1.

(c) Find E[X].

Topic: Probability
Difficulty Level: Medium

Indian Statistical Institute, ISI MMath 2018 Problem 6 Hints along with Solution & Discussion

Hint 1

(a) We use induction on n. Let n=1. The event \{X>1\} means that the first draw yielded a red ball,

    \[ P(X>1)=\frac{1}{2}. \]

Hint 2

(b) If the blue ball is never chosen, then \{X>n\} for all n. Denote by N the event when the blue ball is never chosen. Then

    \[ P(N)\leq P(X>n)=\frac{1}{n+1}\to 0,\ n\to\infty. \]

So, P(N)=0.

Hint 3

(c) For each n\geq 1 we have

    \[ P(X=n)=P(X>n-1)-P(X>n)=\frac{1}{n}-\frac{1}{n+1}=\frac{1}{n(n+1)}. \]

Full Solution

(a) We use induction on n. Let n=1. The event \{X>1\} means that the first draw yielded a red ball,

    \[ P(X>1)=\frac{1}{2}. \]

Assume that event \{X>n\} happens. Then first n draws yielded red balls, and before the (n+1)-st draw there are n+1 red balls and 1 blue ball in the urn. The probability to draw a red ball is \frac{n+1}{n+2}. So,

    \[ P(X>n+1)=P(X>n)P(X>n+1|X>n)=\frac{n+1}{n+2}P(X>n) \]

If P(X>n)=\frac{1}{n+1}, then P(X>n+1)=\frac{1}{n+2}. For n=1 the equality P(X>n)=\frac{1}{n+1} is true. So, for all n\geq 1

    \[ P(X>n)=\frac{1}{n+1} \]

(b) If the blue ball is never chosen, then \{X>n\} for all n. Denote by N the event when the blue ball is never chosen. Then

    \[ P(N)\leq P(X>n)=\frac{1}{n+1}\to 0,\ n\to\infty. \]

So, P(N)=0.

(c) For each n\geq 1 we have

    \[ P(X=n)=P(X>n-1)-P(X>n)=\frac{1}{n}-\frac{1}{n+1}=\frac{1}{n(n+1)}. \]

Then

    \[ E[X]=\sum^\infty_{n=1}nP(X=n)=\sum^\infty_{n=1}\frac{1}{n+1}=\infty. \]

 

Problem 7.

A real number x_0 is said to be a limit point of a set S\subseteq \mathbb{R} if every

neighbourhood of x_0 contains a point of S other than x_0. Consider the

set

    \[ S=\{0\}\cup \left\{\frac{1}{m}+\frac{1}{n}:m,n\in\mathbb{N}\right\} \]

(a) Show that S contains infinitely many limit points of S.

(b) Show that S is a compact subset of \mathbb{R}.

(c) Find all limit points of S.

Topic: Real Analysis
Difficulty Level: Medium

Indian Statistical Institute, ISI MMath 2018 Problem 7 Hints along with Solution & Discussion

Hint 1

(a) Observe that \frac{1}{n}=\frac{1}{2n}+\frac{1}{2n}\in S. Let \epsilon>0 and m>\frac{1}{\epsilon}. Then

    \[ \frac{1}{n}<\frac{1}{n}+\frac{1}{m}<\frac{1}{n}+\epsilon. \]

Hint 2

(b) The set S is bounded: S\subset [0,2]. It remains to prove that S is closed.

Hint 3

(c) Considerations of previous two parts shows that the limit points of S are \{0\}\cup \{\frac{1}{n}:n\geq 1\}.

Full Solution

(a) Observe that \frac{1}{n}=\frac{1}{2n}+\frac{1}{2n}\in S. Let \epsilon>0 and m>\frac{1}{\epsilon}. Then

    \[ \frac{1}{n}<\frac{1}{n}+\frac{1}{m}<\frac{1}{n}+\epsilon. \]

Every neighborhood of \frac{1}{n} contains a point from S distinct from \frac{1}{n}. Each point \frac{1}{n} is a limit point of the set S that belongs to the set S.

 

If m>\frac{2}{\epsilon}, then \frac{1}{m}+\frac{1}{m} is a point of S in the \epsilon-neighborhood of 0. 0 is a limit point of S. We’ve proved that points

    \[ \{0\}\cup \left\{\frac{1}{n}:n\geq 1\right\} \]

are limit points of S.
(b)

The set S is bounded: S\subset [0,2]. It remains to prove that S is closed. Suppose that x\in \bar{S}\setminus S. Every neighborhood of x contains a point y \in S. But x\not\in S and y\ne x. Then x is a limit point of S. Since we assume x\not \in S, it follows that x>0. For every k\geq 1 find a point \frac{1}{m_k}+\frac{1}{n_k}\in S, such that

    \[ \left|x-\left(\frac{1}{m_k}+\frac{1}{n_k}\right)\right|<\frac{1}{k}. \]

If both sequences (m_k)_{k\geq 1} and (n_k)_{k\geq 1} are bounded, then there exist values m,n\geq 1 such that m_k=m and n_k=n for infinitely many values of k. But then x=\frac{1}{m}+\frac{1}{n}\in S, which is not the case. Hence we can assume that (n_k)_{k\geq 1} is unbounded. Let (n_{k_j})_{j\geq 1} be a subsequence such that \lim_{j\to\infty}n_{k_j}= \infty. If the sequence (m_{k_j})_{j\geq 1} is bounded, there exists value m\geq 1 such that m_{k_j}=m for infinitely many values of j. But then

x=\frac{1}{m}\in S. If the sequence (m_{k_j})_{j\geq 1} is unbounded, we can pass to another subsequence (m_{k'_j})_{j\geq 1} that converges to \infty. Then we have x=0. In any case x\in S and S is compact.

(c) Considerations of previous two parts shows that the limit points of S are \{0\}\cup \{\frac{1}{n}:n\geq 1\}.

Problem 8.

(a) Let \{f_n\}^\infty_{n=1} be a sequence of continuous functions on [0, 1] such

that \sum^\infty_{n=1} f_n converges uniformly on (0, 1]. Show that \sum^\infty_{n=1}f_n(0)

converges.

(b) Find the set D of all points x\in [0,1] such that the series

    \[ \sum^\infty_{n=1}e^{-nx}\cos(nx) \]

converges. Does this series converge uniformly on D? Justify your answer.

Topic: Real Analysis
Difficulty Level: Medium

Indian Statistical Institute, ISI MMath 2018 Problem 8 Hints along with Solution & Discussion

Hint 1

(a) Denote s_m(x)=\sum^m_{n=1}f_n(x). The sequence (s_m)_{m\geq 1} converges uniformly on (0,1]: there exists function s on (0,1] such that

    \[ \lim_{m\to\infty}\sup_{0<x\leq 1}|s_m(x)-s(x)|=0. \]

Hint 2

(a) Hint Continued: Verify that \lim_{x\to 0}s(x) exists. Let \epsilon>0. Find m\geq 1 such that

    \[ \sup_{0<x\leq 1}|s_m(x)-s(x)|<\frac{\epsilon}{3}. \]

Hint 3

(b) The series converges for all x\in (0,1]. Indeed,

    \[ \sum^\infty_{n=1}|e^{-nx}\cos(nx)|\leq \sum^\infty_{n=1}e^{-nx}=\frac{e^{-x}}{1-e^{-x}}<\infty. \]

Full Solution

(a) Denote s_m(x)=\sum^m_{n=1}f_n(x). The sequence (s_m)_{m\geq 1} converges uniformly on (0,1]: there exists function s on (0,1] such that

    \[ \lim_{m\to\infty}\sup_{0<x\leq 1}|s_m(x)-s(x)|=0. \]

We verify that \lim_{x\to 0}s(x) exists. Let \epsilon>0. Find m\geq 1 such that

    \[ \sup_{0<x\leq 1}|s_m(x)-s(x)|<\frac{\epsilon}{3}. \]

Using continuity of s_m find \delta>0 such that for all x,y\in (0,\delta)

    \[ |s_m(x)-s_m(y)|<\frac{\epsilon}{3}. \]

Then for all x,y\in (0,\delta) we have

    \[ |s(x)-s(y)|\leq |s(x)-s_m(x)|+|s_m(x)-s_m(y)|+|s_m(y)-s(y)|<\epsilon. \]

Since \epsilon>0 is arbitrary, this shows that \lim_{x\to 0}s(x) exists. We denote the limit by s(0).

 

Now, let \epsilon>0. Find m_0\geq 1 such that for all m\geq m_0 and all x\in(0,1] we have

    \[ |s_m(x)-s(x)|\leq \epsilon. \]

Passing to the limit as x\to 0 we get

    \[ |s_m(0)-s(0)|\leq \epsilon. \]

Since \epsilon>0 is arbitrary, this shows that

    \[ s(0)=\lim_{m\to\infty}s_m(0)=\lim_{m\to\infty}\sum^m_{n=1}f_n(0), \]

i.e. \sum^\infty_{n=1}f_n(0)

converges and has the sum s(0).

(b) The series converges for all x\in (0,1]. Indeed,

    \[ \sum^\infty_{n=1}|e^{-nx}\cos(nx)|\leq \sum^\infty_{n=1}e^{-nx}=\frac{e^{-x}}{1-e^{-x}}<\infty. \]

For x=0 the series diverges, as it is the series 1+1+1+\ldots. So, D=(0,1]. If the series converges uniformly, then it should also converge at x=0. So, the series \sum^\infty_{n=1}e^{-nx}\cos(nx) does not converge uniformly on D.

To view ISI MMATH 2020 solutions, hints, and discussions: Click Here

To view ISI MMATH 2019 solutions, hints, and discussions: Click Here

To view ISI MMATH 2017 solutions, hints, and discussions: Click Here

To view ISI MMATH 2016 solutions, hints, and discussions: Click Here

To view previous year’s question papers: Click Here

4 Comments

  • Sir,my daughter wants to prepare for m.math entrance examination..can you please help her with your classes and practice sets??

  • For Problem 2, Can I solve this like this?

    Let x \in [0,1] then by using MVT there exists t_1 \in (0,x) such that f(x) - f(0) = f'(t_1)x i.e |f(x)| = |f'(t_1)x| \leq c|f(t_1)|x. Similary there exists t_2 \in (0,t_1) such that f(t_1) = f'(t_2)t_1 and applying modulus we get |f(t_1)| = |f'(t_2)|t_1 \leq c|ft_2)|t_1 So with the above inequalities we have |f(x)| \leq c^2 |f(t_2)|t_1x \leq c^2|f(t_2)|. Proceeding this way we have |f(x)| \leq c^n |f(x_n)| but since f is continuous on [0,1] so is |f| so it attains the maximum say M then the inequality becomes |f(x)| \leq  c^n|f(t_n)|\leq c^n M and we know that c^n M \to 0 . so for any \epsilon>0 there exists n_0 \in \mathbb{N} such that for all n \geq n_0 we have c^n M < \epsilon In particular c^{n_0}M < \epsilon hence |f(x)| \leq c^{n_0}M \leq \epsilon this implies f(x)=0 and since x was arbitrary we have f(x) = 0 \forall x \in [0,1]

  • For Problem 2, Can I solve this like this?

    Let x \in [0,1] then by using MVT there exists t_1 \in (0,x) such that f(x) - f(0) = f'(t_1)x i.e |f(x)| = |f'(t_1)x| \leq c|f(t_1)|x. Similary there exists t_2 \in (0,t_1) such that f(t_1) = f'(t_2)t_1 and applying modulus we get |f(t_1)| = |f'(t_2)|t_1 \leq c|ft_2)|t_1 So with the above inequalities we have |f(x)| \leq c^2 |f(t_2)|t_1x \leq c^2|f(t_2)|. Proceeding this way we have |f(x)| \leq c^n |f(x_n)| but since f is continuous on [0,1] so is |f| so it attains the maximum say M then the inequality becomes |f(x)| \leq  c^n|f(t_n)|\leq c^n M and we know that c^n M \to 0 . so for any \epsilon>0 there exists n_0 \in \mathbb{N} such that for all n \geq n_0 we have c^n M < \epsilon In particular c^{n_0}M < \epsilon hence |f(x)| \leq c^{n_0}M \leq \epsilon this implies f(x)=0 and since x was arbitrary we have f(x) = 0 \forall x \in [0,1]

Leave a Reply to Arkabrata Das Cancel reply